Contracts 1 - MC (Review for Final Exam)

Ace your homework & exams now with Quizwiz!

G, a general contractor, solicited bids for a project from subcontractors, including S. S replied "I can't give you a bid, because I don't trust your specifications. But it should be about $2.00 per cubic foot. It's fine with me if you want to use my quote or if you don't." G used the quote from S in preparing her bid, and was awarded the contract. Because of this, G claims that S has a duty to perform at a rate of $2.00 per cubic foot. Is the quote from S an offer? A)Because S was responding to a request for a bid, S's response will be treated as an offer. B) Because S's response included a specific price, S's response will be treated as an offer. C) Because S's response included the words "I can't give you a bid," S's response is not an offer. D) Because G relied on S's response in making her bid, S's response will be treated as an offer.

C -- Section 2.4 of Hornbook.

Which of the following statements regarding estimates is most accurate? A) An estimate can never be an offer. B) An estimate will be an offer unless it clearly states that it is intended as an estimate only. C) If the communication contains the word "estimate" in can never be treated as an offer. D) An estimate can be an offer if it is sent in response to an invitation to bid.

D -- Section 2.6(b) of Hornbook.

The power of acceptance: A) can be exercised with or without knowledge of the offer. B) can be exercised only with knowledge of the offer. C) is created when the offeree agrees to the offer. D) is created when the offeror certifies that the offeree has accepted the offer.

B -- the offeree must have knowledge of the offer in order to accept it, which is why (a) is incorrect. (c) and (d) are incorrect, because the power of acceptance is created when the offeror makes a valid offer. See Broadnax v. Ledbetter.

A, a merchant, makes an offer by mail to B in a signed writing. The offer provides assurances that it will be held open for two weeks. B sends an unequivocal acceptance by mail to A's authorized address. The contract is formed when: A) B first manifests acceptance. B) B puts the correspondence in the mail box. C) B's correspondence is received by A. D) B's correspondence is read by A.

C -- A merchant's firm offer creates an EXCEPTION to the mailbox rule; during the period in which the offer is being held open under a merchant's firm offer, acceptance is effective upon receipt, which is why (b) is incorrect. (a) is incorrect, communication of B's manifestation is required for it to be effective. (d) is incorrect, as the acceptance is effective upon receipt, whether the offeror reads it or not. See Hornbook Section 2.25(d).

A submits an offer in writing to provide B with a service. B quickly responds in writing stating that she accepts the offer, but includes somewhat different terms. Most likely B's response is: A) an acceptance. B) a counteroffer. C) a counteroffer and rejection of the offer. D) a rejection.

C -- At common law, B has created a new offer, which acts as a counter offer; without expressly keeping the old offer open, the counter offer will also revoke the prior offer, which is why (b) and (d) are incorrect. (a) is incorrect, most likely B's response will not be an acceptance. See Hornbook Section 2.21(a).

A pays B $100 in exchange for B's promise to keep his offer to provide childcare open for six months. Can A enforce B's promise to keep the childcare offer open for six months? A) Yes, because there is consideration to support B's promise. B) Yes, because the option proposes a fair exchange within a reasonable time. C) No, because it's not in writing. D) No, because the time period of the option exceeds three months.

A -- A promise supported by consideration (or a consideration substitute) creates an enforceable option contract. (b) is incorrect, the basis for enforcing an option contract is consideration (or a consideration substitute). (c) is incorrect, options are not required to be in writing to be enforceable. (d) is incorrect, options are not limited to three months. See Hornbook Section 2.25.

What is meant by the "objective theory of contracts?" A) Communications by one party are to be interpreted from the vantage point of a reasonable person in the position of the other party. B) Communications by one party are to be interpreted according to the intent of the party making the communication. C) To be valid a contract must have a clear "objective" that is sufficiently stated in the potential offer. D) If the objectives of a contract are not fully identified in an offer the offer will fail for indefiniteness.

A -- Section 2.2 of Hornbook.

Which of the following is not an important factor in determining whether a communication is or is not an offer? A) The manner used to communicate the potential offer. B) Usage of the trade. C) The relationship of the parties. D) Surrounding circumstances.

A -- Section 2.6(g) of Hornbook.

Harry and Sandra have been happily married since 1866. Harry needs to go to England to write a play and before he leaves, in order to maintain domestic tranquility, he agrees to pay Sandra $5000 per week for her support. They both agree that Sandra has financial needs and that she has always worked hard at maintaining the house.Has an enforceable contract been formed? A) No. There is a presumption that the parties did not intend to create a contract. B) Yes, because the parties agreed that Sandra has financial needs and Harry agreed to pay. C) Yes, if Harry has the ability to pay. D) No, because Harry has no obligation to support his wife.

A -- The general rule applicable is that if the parties do not intend to be bound or do not intend legal consequences, there will be no contract. There was a presumption in the 19th century (and perhaps even today) that when a husband and wife are living together amicably, an agreement between them with respect to a housekeeping allowance will not be enforced as a contract. (b) and (c) are incorrect, as they are not part of the requirements for forming a contract; (d) is incorrect as it misstates the law. If you missed this question, please review Section 2.5 of the Hornbook (6th Edition) page 27; page 29 (7th Edition).

Assume that an offeree learns from a reliable source that land that has been offered to her by a seller, has also been offered by the seller to another interested party who had not yet accepted. Which one of the following statements most accurately describes the offeree's rights? A) The offeree may accept the offer. B) The offeree may accept the offer but only after a reasonable inquiry into the information's accuracy, C) The offeree may not accept the offer because it would expose the offeror to potential multiple liability. D) The offeree may not accept the offer because by making duplicate identical offers, the offer to offeree has effectively been revoked.

A -- The offer is still open and the offeree has the power of acceptance; the fact that another party also has the power to accept does not prevent the offeree from accepting. (b) is incorrect, no such inquiry is required. (c) is incorrect, the potential for such exposure does not restrict the offeree's power of acceptance, since the offeror is the one that created the situation in the first place. (d) is incorrect, granting the power to accept to multiple offerees does not revoke the offer. See Hornbook Section 2.20(d).

A submitted an offer in writing to provide B with goods. B quickly responded in writing stating that she accepted A's offer, but B's written response includes some additional terms. A court will most likely find that B's response is A) an acceptance. B) a counter offer. C) a counter offer and rejection. D) a rejection.

A -- Under UCC 2-207(1), "A definite and seasonable expression of acceptance or a written confirmation which is sent within a reasonable time operates as an acceptance even though it states terms additional to or different from those offered or agreed upon,...", which is why (b), (c), and (d) are incorrect. See Hornbook Section 2.21(b).

A submits an offer in writing to provide B with goods and B responds in writing stating that she accepts the offer, but B's response includes additional material terms. If A provides the goods and B accepts them, which of the following best describes the contractual relationship between A and B? A) A contract including only A's terms. B) A contract including B's terms. C) No contract. D) A contract only including the terms found in both the A and B's writings.

A -- Under the UCC 2-207(2), since there is nothing to indicate that the parties are merchants, a contract would be formed, but it would not include the additional terms in B's acceptance, which is why (b), (c), and (d) are incorrect. See Hornbook Section 2.21(b).

Each provision of the Uniform Commercial Code is followed by official comments. What is the legal effect of the official comments? A) While most states have enacted into law the provisions of the UCC, most have not enacted into law the official comments. B) Most states have enacted into law both the provisions of the UCC and the official comments. C) Most courts will not consult the official comments for guidance since they have not been enacted into law in most jurisdictions. D) The official comment carry the same legal authority within a particular jurisdiction is do the provisions of the UCC.

A -- accurately states the status of the official comments (not enacted into law)

Which of the following is the most accurate statement as to the role of a judge and jury in a court of law? A) A judge is to determine questions of law and a jury is to determine questions of fact. B) A judge is to determine questions of fact and a jury is to determine questions of law. C) A jury is to decide questions of law and fact, but a judge is empowered to override the jury's determination. D) A judge is to decide questions of law and fact based on the recommendations of the jury.

A -- as it accurately states the roles of a judge and a jury in a jury trial (note: when a judge sits without a jury, the judge determines both questions of law and questions of fact).

Which of the following statements is NOT true? A) A "bad deal" on the part of a party to a contract will render consideration inadequate. B) It would be an unwarranted interference with freedom of contract if a court were to relieve an adult party from a bad exchange. C) Generally, courts do not review the adequacy of the consideration of a contract. D) Economic inadequacy of a detriment is one of the factors courts will consider in determining whether the promisor really exchanged a promise in return for a small detriment.

A -- as it is not true; that a party received a "bad deal" does not render consideration inadequate. (b), (c), and (d) are all incorrect answers, as they are all true statements. See Hornbook Section 4.4.

Jeff does fine carpentry work. Mutt and Jeff enter into a contract whereby Jeff will perform carpentry for Mutt and Mutt will pay $10,000 to Jeff upon completion of the work. Jeff finishes the work and Mutt is so pleased he tells Jeff that he will pay Jeff an additional $1,000. Is Mutt obligated by contract to pay the extra $1,000? A) No, because Mutt's promise to pay the additional $1,000 was not supported by consideration. B) No, because Jeff did not ask for the extra $1,000 prior to completion. C) Yes, because Jeff performed at a level in excess of Mutt's expectations and Mutt promised to pay the additional sum. D) Yes, because Jeff could reasonably believe that Mutt would pay as agreed.

A -- because Mutt's promise was not supported by any new consideration provided by Jeff. Jeff had completed the work and there was nothing else Jeff could provide to Mutt to support Mutt's promise to pay the additional $1,000. Past consideration (Jeff's performance of the services) cannot support a new promise (Mutt's promise to pay an additional $1,000). Moreover, there are no facts to suggest that the increase was a fair and equitable modification in view of circumstances not anticipated by the parties when the contract was entered into. (b), (c), and (d) are incorrect, as none of those factors are relevant to whether Mutt's promise is enforceable; the only issue in determining whether Mutt's promise to pay an additional $1,000 is enforceable is whether it is supported by additional consideration. See Hornbook Section 4.3 (6th Edition) pages 153-154; pages 160-162 (7th Edition).

A says to B, "If you promise to paint my house, I promise to pay you $1500." B starts to paint in A's presence. Is there a contract? A) Yes, because B's conduct constitutes an implied promise to paint A's house. B) No, because A requested a promise and not an act. C) Yes, because B will be entitled to the reasonable value of her services. D) No, because no color, a material term was specified.

A -- because an offer seeking a promise (a bilateral contract offer) may be accepted by either an express or implied promise. An implied promise will be inferred when the offeree begins performance of the act requested in a manner that would reasonably suggest to the offeror that the offeree was in fact promising to perform the act requested, which is why (b) is incorrect. (c) is incorrect, because the reasonable value of a party's services is not part of contract formation. (d) is incorrect, where no color is specified in a contract to paint, the implied color is the present color of the thing to be painted. See Hornbook Section 2.10(a) (6th Edition) page 57; page 62 (7th Edition).

Sarah manufactures and sells novelty dolls. Bernhardt is a retail novelty store owner who specializes in dolls. Sarah and Bernhardt enter into a written agreement wherein Bernhardt agrees to buy from Sarah 600 of Sarah's novelty dolls at a specific price. One particular clause in the agreement states that Bernhardt has the right to purchase an additional 400 dolls at any time during the time period covered by the agreement. Bernhardt orders an additional 400 dolls, but Sarah does not deliver them. What are the rights of the parties? A) Bernhardt has the right to purchase 400 additional dolls from Sarah under the terms of the contract. B) Bernhardt has no right to purchase the 400 additional dolls from Sarah. C) Sarah does not have to deliver the 400 additional dolls unless she and Bernhardt agree on a separate consideration. D) Sarah has to deliver the 400 additional dolls, but a new price must be established.

A -- because the obligation of Sarah to deliver up to 400 additional dolls during the term of the agreement was supported by consideration: Bernhardt's express promise to pay for the original 600 dolls and an implied promise to pay for up to 400 additional dolls on the same terms as the original 600. One consideration (Bernhardt's express promise to pay for the original 600 dolls) can support more than one promise (the promise by Sarah to deliver 600 dolls and then 400 more if requested by Bernhardt), which is why (c) is incorrect. (b) is incorrect, as Bernhardt's contract with Sarah expressly provides for this. (d) is incorrect. Because the provision allowing Bernhardt to purchase up to 400 additional dolls is silent on price, there is an implied term of the same price as the initial 600 dolls. See Hornbook Section 4.15 (6th Edition) page 192; page 200 (7th Edition).

Marie read the following statement in a Stargell's catalog: "Lots available for $20,000. First come, first served. Appear in person, complete application and tender funds." The catalog also stated the physical location of the real estate. Marie went to the physical location, completed the application and tendered $20,000 in cash that was accepted by Stargell. Subsequently, Stargell mailed Marie a cashier's check for $20,000 with a note stating that Marie's application was "rejected."At trial, Marie proved that Stargell's had lots available, but only sold to prominent people who endorse Stargell's company. Does Marie prevail on a claim for breach of contract? A) Yes, because Stargell accepted Marie's offer to purchase the lot according to the terms expressed in Stargell's catalog. B) Yes, because Stargell cannot base acceptance on the basis of status and willingness to endorse. C) No, because the conditions expressed in the catalog did not constitute an offer which Marie could accept. D) No, because a reasonable person should know that she cannot rely on statements in a catalog.

A -- is correct, because, although statements in catalogs are generally not considered offers, but are instead invitations for offers, when Marie appeared, completed the application and paid the lot fee, her offer implicitly included the conditions expressed in the catalog. When Marie proved that Stargell failed to comply with the criteria in the catalog, she stated a cause of action for breach of contract. (c) is incorrect because the conditions expressed were an offer; (b) is incorrect because acceptance can be based on such factors, if so stated; (d) is incorrect, because it has no bearing on whether or not the conditions expressed were an offer. If you missed this question, please review Section 2.6(d) of the Hornbook (6th Edition) page 33; page 36 (7th Edition).

Which of the following statements best describes the Restatement of Contracts? A) It does not have the force of law, but is highly persuasive authority. B) It is written by the American Law Institute, which is a legislative body charged with codifying the law of contracts. C) It is a restatement of the law contained in the Uniform Commercial Code. D) It was co-written by Professors Williston and Corbin, two leading contract scholars.

A -- page 14 hornbook

For public offers and rewards, the offer: A) can be accepted by the means specified in the offer. B) can be accepted by any means that is reasonable under the circumstances. C) cannot be accepted, if it is in the form of an advertisement. D) cannot be accepted, unless it is in the form of an advertisement.

A -- public offers and rewards can be accepted by the means specified in the offer, which is which is why (b) is incorrect. (c) and (d) are incorrect, whether the public offer or reward is or is not in the form of an advertisement is irrelevant to its ability to serve as a valid offer. See Carlill v. Carbolic Smoke Ball Co. and Leonard v. Pepsico.

If two parties have entered into a contract for the sale of goods that is complete in all material respects but lacks any express language identifying the time and place of payment, a court would likely rule that the gap: A) should be filled with a gap-filler provision of the UCC. B) should be filled based on the standard practices in the relevant trade or profession. C) should be filled based on what would be reasonable. D) would make the contract unenforceable for indefiniteness.

A -- the UCC provides specific gap-fillers for terms such as time and place of payment. (b) and (c) are incorrect, as the UCC's expressly stated gap-fillers control over any analysis based on reasonableness or trade standards. (d) is incorrect, as UCC 2-204(3) would prevent the contract from being unenforceable where the intent to form a contract is clear. See Southwest Engineering Co. v. Martin Tractor Co.

Under the UCC Article 2, a written modification is required in which one of the following situations? A) If the contract as modified is within the Statute of Frauds provision of the UCC . B) If the contract as originally written is within the Statute of Frauds provision of the UCC. C) A written modification is always required. D) A written modification is never required so long as both parties agree to the new term(s).

A -- the UCC requires a writing to modify a contract where that contract as modified is within the Statute of Frauds, this is why (d) is incorrect. (b) is incorrect, the UCC applies this writing requirement to the proposed change, not to the original version of the contract. (c) is incorrect, the UCC does not always requires a writing to modify a contract. See Hornbook Section 5.14.

A agrees to remodel B's home pursuant to specific plans and specifications. The price for the job is $8,500. The plans do not specify the materials that A must use. Is the contract void? A) Yes, if the omission of the terms describing the materials to be used cannot be implied due to lack of objective standards. B) No, because the courts may imply a missing term once the parties have agreed on the price and subject matter of the contract. C) Yes, contracts are required to have the specific materials be disclosed. D) No, when no materials are specified, a builder may use materials that are reasonable under the circumstances.

A -- the best answer, because the courts will not imply a missing term in a contract if the court lacks objective standards to determine what term would be appropriate to fill the gap in the agreement of the parties; this answer choice addresses a fundamental requirement that can prevent a contract from being formed. (b) is incorrect, because the courts will not imply a missing term in a contract if the court lacks objective standards to determine what term would be appropriate to fill the gap in the agreement of the parties. (c) is incorrect, as there is no requirement that the contract must specify the materials to be used. (d) is not incorrect (as this is the standard that will guide the builder's action), but it is not the best answer among the choices offered, because it assumes that a valid contract has been formed as opposed to addressing the more fundamental issue, as seen in (a), of whether the missing term will prevent a contract from being formed. See Hornbook (6th Edition) page 46-48; pages 51-53 (7th Edition).

A owes B $20,000 and A agrees that he will pay back the money to B "so long as my union does not order me to walk off my job." Later, A's union goes on strike and orders A to walk off the job. Does A have an obligation to pay B the $20,000 owed? A) No, A does not have to pay B because A's promise to pay so long as A is not ordered by his union to walk off the job is outside the control of A. B) Yes, A has to pay B because his promise in illusory. C) No, A does not have to pay B because walking off the job when ordered by the union was part of the agreement. D) Yes, A has to pay B because A obviously knew he was going to go on strike or he would not have included that language in the agreement.

A -- the condition ("so long as my union does not order me to walk off my job") is an enforceable condition because it is beyond the control of A, meaning that it is not illusory, which is why (b) is incorrect. (c) is correct but not the best answer, because it does not explain the law relating to the condition in the contract as completely as (a). (d) is incorrect, as there is no information on this point and it is not relevant to the analysis of the condition (since the occurrence of the condition is beyond A's control). See Hornbook Section 4.12(b)(6).

See Hornbook Section 2.9(b).Under the UCC, the question of whether the parties intended to contract: A) will ordinarily be a question of fact. B) will ordinarily be a question of law. C) is always determined by the parties' previous course of dealing. D) is always determined by the practices of the trade or industry in question.

A -- the court ordinarily must make a factual determination of the intent of the parties. (b) is incorrect, see (a). (c) is incorrect, not only is the standard proposed inaccurate, it would also be useless for resolving this question between parties that had never contracted before. (d) is incorrect, as it prevents the parties to the contract in question from being able to determine whether they intended to form a contract or not.

Modification of an enforceable agreement requires new consideration A) if the contract is governed by the common law. B) if the contract is governed by the UCC Article 2. C) if the modification is not in writing. D) always because consideration is always required for all modifications.

A -- under the common law modification of an enforceable agreement requires new consideration. (b) and (d) are incorrect, as contracts governed by the UCC do not require consideration for modification. (c) is incorrect, whether a contract is oral or in writing has no effect on whether consideration is required for modification. See Hornbook Section 5.14.

Buyer telephoned seller and made the seller a valid offer to buy goods from the seller. Assume that the seller did not accept the offer during the telephone call. Absent any manifestation of contrary intent by either party, at what time will most courts find that the buyer's offer lapsed? A) After the phone call came to an end. B) 24 hours after the phone call ended. C) 10 days after the phone call ended. D) When the offeror subjectively intended it to lapse.

A -- unless there is a manifestation of intent to the contrary, the offer will lapse (terminate) when the phone call comes to an end (because that is a reasonable amount of time under the circumstances). (b) and (c) are incorrect, neither party expressed such an intent, which would be required for the offer to remain open for either period of time. (d) is incorrect, because the offeree's subjective intent does not control the duration of an offer. See Hornbook Section 2.20(a).

Define Condition

An act or event that affects a party's contractual duty. It is a qualification that is placed on an obligation.

Marta read this statement on Developer's website: "Condominiums available for $120,000. First come, first served. Appear in person at our office, complete our application, and tender 10% down payment." The website included the physical location of the condominium and the unit numbers of the condominiums that were available. Marta went to Developer's office, completed the application and tendered $12,000 in cash that was accepted by Developer. A few days later, Developer mailed Marta a letter informing her that her application had been rejected. Enclosed with the letter was a cashier's check for $12,000. At trial, Marta proved that Developer had units available for sale, but only sold to prominent people who would endorse Developer's projects. Can Marta prevail on a claim for breach of contract? A) No, because the conditions expressed on the website did not constitute an offer which Marta could accept. B) Yes, because Developer accepted Marta's offer to purchase the condominium according to the terms stated on Developer's website. C) Yes, because Developer cannot base acceptance on the basis of status and willingness to endorse. D) No, because a reasonable person should know that she cannot rely on statements on a website.

B -- Although statements on websites are generally not considered offers, but are instead invitations for offers, when Marta appeared, completed the application and paid the down payment, her offer implicitly included the conditions expressed on the website. When Marta proved that Developer failed to comply with the criteria listed on its website, she stated a cause of action for breach of contract. (a) is incorrect because the conditions expressed were an offer; (c) is incorrect because acceptance can be based on such factors, if so stated; (d) is incorrect, because it has no bearing on whether or not the conditions expressed were an offer. See Section 2.6(d) of the Hornbook (6th Edition) page 33; page 36 (7th Edition).

Barry signed an agreement without reading it, but checked off a box in the agreement that stated: "By checking this box, you agree that you have read and understood the entire agreement and that all terms contained therein are bargained for." Which of the following statements most accurately states the legal effect of Barry's conduct? A) Since Barry really didn't bargain for anything in the agreement, there is no consideration. B) Even though Barry did not read the agreement, if the terms objectively contain the appearance of a bargained for exchange, consideration is present. C) Even though it was foolish for Barry to check box without reading the agreement, it is a likely occurrence so a court will not require Barry to perform the agreement. D) It is against public policy to enforce a check-off like this if Barry did not, in fact, read the agreement.

B -- If, when viewed objectively, the terms when contain the appearance of a bargained-for exchange, then the fact that Barry did not actually read the agreement is irrelevant. That Barry did not actually read the terms provides him with no relief or protection, which is why (a), (c), and (d) are incorrect. See Hornbook Section 4.2(d).

Review the following conversation:Kathy: "Jerry, will you consider buying my cabin in Aspen? If you will make me an offer, I will consider it."Jerry: "Okay, $5.00."Kathy: "No, that is not enough."Jerry: "Will you accept $85,000?"Kathy: "I won't sell it for less than $95,000."Jerry: "I accept."Was there a contract? A) Yes, because Jerry accepted Kathy's offer to sell the property for $95,000. B) No, because Kathy never offered to sell the property for $95,000. C) No, because Kathy did not have a reasonable time within which to revoke. D) Yes, Kathy intended to sell the house for $95,000.

B -- Kathy stated that she would consider an offer made by Jerry, not that Kathy was making Jerry an offer to sell the property. When Jerry asked for a price, Kathy responded with a price quotation. However, Kathy never made an offer to sell the property to Jerry. (a) is incorrect, because Kathy never made an offer to sell the property to Jerry. (b) is incorrect, having a reasonable time within which to revoke is not required for an offer to be valid (or for an acceptance to be valid). (d) is incorrect, Kathy's intent to form a contract is not clear (which is what prevented Kathy from making a valid offer). See Hornbook (6th Edition) pages 38-40; pages 41-43 (7th Edition).

City posted this announcement on its website: "The City will pay $2,000 to anyone with information leading to the arrest and conviction of the person or persons responsible for vandalizing City Park last weekend." Anne saw it all happen, and knows the names of the people who did it. If she wants to receive the reward: A) before doing anything else, Anne must first inform City that she has the information. B) Anne can accept the offer by providing the information to City. C) Anne can accept the offer by promising to provide the information to City. D) Anne must make an offer to provide the information, as City's announcement is merely an inquiry or a solicitation that does not give anyone the power of acceptance.

B -- Rewards are offers for unilateral contracts, and can only be accepted by performance. (a) is incorrect, as there is no requirement that notice of the intent to perform be given before accepting by performance where the offeror will become aware of the performance in the normal course of that performance; (c) is incorrect, because rewards are not offers for bilateral contracts, and cannot be accepted by promising to perform; (d) is incorrect, because City's announcement has required information in sufficient detail to function as a valid offer.

Pearl has lost her prize-winning cat, Mr. Whiskers. On the website where she chronicles the competitions in which she enters Mr. Whiskers, Pearl offers a $2,000 reward for the return of her beloved cat, which (elsewhere on the website) she claims is worth at least $10,000. When Diego returns the cat, Pearl refuses to pay. When Diego sues to recover the $2,000, it is most likely that Diego will: A) prevail, because rewards are always enforceable. B) prevail, if a reasonable person would conclude from the statement on Pearl's website that she intended to enter into a contract. C) not prevail, because there was no meeting of the minds and Pearl did not intend to enter into a contract. D) not prevail, because Diego knew or should have known that Pearl was overstating the value of the cat.

B -- The "objective theory" of contracts (the majority rule) is that the subjective intent of the parties is not relevant when determining whether a party has given assent to enter into a contract. This principle is true regardless of what the party actually intended. (a) is incorrect, because rewards are not always enforceable, they still have to meet the requirements for a valid offer, regardless of whether they are in the form of a reward.

Indefinite contract terms: A) are always unenforceable. B) can in some instances be understood by applying the duty to perform with reasonable effort and in good faith. C) will be gap-filled. D) will be gap-filled, but only if the UCC applies to the contract.

B -- the duty to perform with reasonable effort and in good faith (implied in all contracts) can in some instances be used to determine the nature of the performance required by the party. (a) is incorrect, see (b). (c) is incorrect, because gap-fillers are used where there is no term expressed, not to replace an indefinite term. (d) is incorrect, because (1) gap-fillers are used where there is no term expressed, not to replace an indefinite term, and (2) the common law also has gap-filling provisions. See Wood v. Lucy, Lady Duff-Gordon and Hornbook Section 2.9(a)(2).

Max and Sam have been business partners for years, with Max being the managing partner and Sam being a passive investor. Max needs to go to Canada to settle the estate of his uncle and before he leaves, in order to keep the business open, Max agrees to pay Sam $20,000 per month for his services managing the business while Max is away. They both agree that the business cannot run itself, and Sam needs to protect both of their investments. Has an enforceable contract been formed? A) Yes, because the parties agreed that the business cannot run itself. B) No, if there is a presumption that the parties did not intend to create a contract. C) No, because Max has no obligation to pay Sam for his services. D) Yes, if Max has the ability to pay.

B -- The general rule applicable is that if the parties do not intend to be bound or do not intend legal consequences, there will be NO contract. (a) is incorrect, because an agreement regarding the presence of a need is not the same as an intent to form a contract. (c) is incorrect, because this contract is not about the enforcement of "extra-contractual" obligations; rather it is an independent obligation and the focus is whether the obligation at the center of this possible agreement is an enforceable contract. (d) is incorrect, because the ability to pay of one of the parties is not one of the elements required for the formation of a valid contract, and it has no impact on any of those elements. See Section 2.5 of the Hornbook (6th Edition) page 27; page 29 (7th Edition).

Assume that A and B are both merchants and they have an oral agreement for the sale of goods. Assume also that B quickly sent a written confirmation of the oral agreement but the confirmation contained additional non-material terms. If, after several weeks, A has not objected, how would a court most likely rule? A) A contract exists between A and B including only A's terms. B) A contract exists between A and B including B's terms. C) No contract exists between A and B. D) A contract exists between A and B but only including those terms found in both the A and B writings.

B -- Under UCC 2-207(2), "...Between merchants such terms become part of the contract unless:... (c) notification of objection to them has already been given or is given within a reasonable time after notice of them is received." Which is why (a), (c), and (d) are incorrect. See Hornbook Section 2.21(b).

A seller located in China makes an offer by mail for the sale of goods to a buyer located in the U.S. Assume the buyer promptly mailed an acceptance to the seller and that both parties are merchants. When would the contract be formed? A) When the acceptance is dispatched. B) When the acceptance is received. C) When the acceptance is read. D) When the acceptance is received, but only after the offeror has given the offeree notice of receipt.

B -- Under the United Nations Convention for the International Sale of Goods (which would apply to this international transaction), acceptances are effective upon receipt, which is why (a) is incorrect. (c) is incorrect, the acceptance is effective upon receipt, whether the offeror reads it or not. (d) is incorrect, there is no requirement that the offeror must give notice of receipt to the offeree for the acceptance to be effective. See Hornbook Section 2.26.

A submitted an offer in writing to provide B with a service. B responded in writing stating that she accepts the offer, but B included somewhat different material terms. If A begins to provide the agreed services for B, a court will most likely rule that there is: A) A contract including only the terms in A's offer. B) A contract including the terms in B's written response. C) No contract between A and B. D) A contract including only the terms found in both A and B's writings.

B -- Under the common law "last shot rule," since A is performing without objection after receiving B's terms, the last articulation of the terms by B will be enforceable, which is why (a) is incorrect. (c) is incorrect, it is clear from the conduct of the parties that a contract has been formed. (d) is incorrect, as it misstates the last shot rule. See Hornbook Section 2.21(a).

Sandy says to Dave, "I promise to buy you lunch if you pay me the $100 you owe me." Which of the following statements is most accurate regarding Sandy's promise? A) Sandy's promise is illusory and is not consideration. B) Sandy's promise is not consideration because Dave's performance would merely be performance of something Dave was already legally obligated to do. C) Sandy's promise is a "sham" and is not consideration. D) Sandy's promise is consideration

B -- While Sandy's promise it could serve as consideration under other circumstances (which is why (a) and (c) are incorrect), it cannot serve as consideration under these circumstances, where it is an inducement to get Dave to perform what he is already obligated to do (which is why (d) is not correct). See Hornbook Section 4.12(a).

A makes an offer to B, suggesting no particular means of acceptance. B responds by mail. Assuming a response by mail is reasonable, when is B's acceptance effective? A) B first manifests acceptance. B) When put in the mail box. C) When received by A. D) When read by A.

B -- While the general rule for business communications is that they are effective upon receipt, under the "mailbox rule" acceptances are effective upon dispatch (when put in the box), which is why (c) is incorrect. (a) is incorrect, the offeror's manifestation of intent must be communicated to the offeree to take effect. (d) is incorrect, as the acceptance will be effective upon dispatch, whether the offeree reads it or not. See Hornbook Section 2.23(a).

O was the owner of vacant land in another state. When he visited the land, he found that S had built a house on the empty lot. When O tried to enter the house, S told O that if O stayed there, O would be contractually obligated to pay for the house. Is S correct? A) Yes, because O's conduct of using the house would constitute an acceptance of S's offer. B) No, because O had no opportunity to reject the house. C) Yes, because one who benefits from the service of another must pay the reasonable value of the service. D) No, because S was a trespasser and cannot gain any value from being there.

B -- because silence will generally not constitute acceptance of an offer. If an offeree accepts the benefits of the offeror's services, the silence of the offeree may sometimes be sufficient to constitute an acceptance. However, an acceptance by silence will only be found when the offeree has remained silent even after a reasonable opportunity to reject the goods or services offered by the offeror. Here, O had no opportunity to reject the offer because he was in another state and unaware of the construction of the house until after it was completed, which is why (a) is incorrect. (c) is incorrect, the principle that it states is not part of contract formation. (d) is incorrect, because a trespasser (under certain circumstances) can recover the value added by making improvements to the property. See Hornbook Section 2.18 (6th Edition) pages 71-75; pages 77-81 (7th Edition).

When Darlene bought some mountain property, she constructed a road on her property, but next to the border of adjacent land owned by Ms. Painter. Darlene asked Ms. Painter if she would be interested in sharing the cost of construction and maintenance in exchange for use of the road. They agreed. By the terms of their written agreement, Ms. Painter was allowed to terminate the agreement at will. Everything went fine for a number of years and both Darlene and Ms. Painter paid one-half of the road maintenance expense until Darlene refused to allow Ms. Painter to use the road. Ms. Painter sues Darlene for breach of contract and Darlene claims, as her defense, mutuality of agreement. What result? A) Darlene will prevail, because Ms. Painter had no legal obligation under the contract. B) Ms. Painter will prevail, because the court would imply that reasonable notice be given prior to termination. C) Darlene will prevail, because there was no time specified in the agreement. D) Ms. Painter will prevail, because there was no time specified in the agreement.

B -- because the courts will imply a reasonable notice requirement in such contracts to avoid finding that the contract is unenforceable due to a "termination at will" clause. Thus, the court would imply that either party must provide reasonable notice before terminating the contract, which is why (a) is incorrect. (c) and (d) are incorrect; that no time was specified in the termination at will clause is the reason for the court implying a reasonable notice requirement. See Hornbook Section 4.12(b)(4-5) (6th Edition) pages 179-183; pages 187-191 (7th Edition).

Davis, a supplier of religious items, needs to obtain an inventory of metal crosses. He contracts with Petterman to make and deliver 10,000 crosses at $1.00 per cross. Petterman manufactures and delivers half of the crosses and Davis pays $5,000. Petterman then tells Davis that the price has gone up to $1.50 per cross. Davis agrees because he needs the crosses. When Petterman delivers the crosses, what price is she entitled to be paid? A) At a rate of $1.00 each, because there was no consideration to support Davis' promise to pay $1.50 each. B) At a rate of $1.50 each, if the modification was requested in good faith. C) At a rate of $1.50 each, because Petterman is the master of the offer. D) At a rate of $1.00 each, because one cannot change the price after performance has begun.

B -- because this contract is governed by Article 2 of the Uniform Commercial Code (since the contract involves the sale of goods). Under UCC 2-209, an agreement modifying a contract for the sale of goods needs no consideration to be binding (as long as the modification was requested in good faith), which is why (a) is incorrect. (c) is incorrect, because the offeror does not have the ability to unilaterally alter the terms of a contract. (d) is incorrect, as contracts can be modified; under the UCC, this can be accomplished by meeting the requirement of UCC 2-209 that the modification be requested in good faith. See Hornbook Section 5.41(a) (6th Edition) pages 209-211; pages 218-219 (7th Edition).

Mother says to Sissy, "since you have taken care of me all these years, I promise to pay you $100,000 at the end of the year." Will a court enforce Mother's promise to Sissy? A) No, Mother does not have to pay Sissy because Mother may not live until the end of the year. B) No, Mother does not have to pay because Mother's promise did not induce any detriment by Sissy. C) Yes, Mother has to pay because Sissy took care of her. D) Yes, Mother has to pay because it was not Sissy's obligation to care for Mother, thus taking care of her was a legal detriment.

B -- generally past actions cannot serve as consideration to support a contract which is why (c) is incorrect. (a) is incorrect, because Mother's survival to the end of the year is not a condition of Mother's promise; rather this is a statement of when the proposed payment would be made. (d) is incorrect, as no contract can be formed based on past actions (however, if Sissy had not yet provided care for Mother, then the analysis would be correct). See Hornbook Section 4.3.

An offeree responds to an offer with a proposal for a similar but different contract, but also notes that the offeree will keep the original offer under advisement. Offeree's proposal is best understood as: A) an offer. B) a counter offer. C) a counteroffer and rejection of the offer. D) a rejection.

B -- offeree has created a new offer while expressly keeping the old offer open, which is why (c) is incorrect. (a) is incorrect, since there is already an offer between the parties, offeree's response creates a counter offer. (d) is incorrect, offeree has been careful not to reject the old offer. See Hornbook Section 2.20(e).

A party's ability to modify an agreement under the UCC requires good faith. Which of the following statements is correct regarding the two distinct inquiries a court must make to determine if the parties entered into the modification in good faith? A) A reasonable person would believe the conduct of the party seeking modification is consistent with "reasonable commercial standards" and whether the party provided consideration. B) The party asserting the modification must demonstrate that he was in fact motivated by a legitimate honest commercial reason and that the conduct is consistent with "reasonable commercial standards of fair dealing in the trade." C) The party asserting the modification demonstrated an attempt to contact the other party and whether the other party was provided an opportunity to be heard. D) Party asserting the modification must demonstrate emergency and that the modification is reasonable under prevailing commercial standards.

B -- party seeking modification must prove both (1) that he was in fact motivated by a legitimate honest commercial reason and (2) that the conduct is consistent with "reasonable commercial standards of fair dealing in the trade," which is why (c) is incorrect. (a) is incorrect, under the UCC consideration is not required for modification. (d) is incorrect, there is no requirement that an emergency must be demonstrated to support a modification. See Roth Steel Products v. Sharon Steel Corp.

Which of the following is the most accurate statement? A) Article 2 of the Uniform Commercial Code represents majority law to be used to determine the outcome of disputes involving any commercial contract. B) Article 2 of the Uniform Commercial Code represents majority law to be used to determine the outcome of disputes involving a contract for the sale of goods. C) Article 2 of the Uniform Commercial Code represents majority law to be used to determine the outcome of disputes involving a contract for the sale of goods, but only if the parties are merchants. D) Article 2 of the Uniform Commercial Code does not represent majority law.

B -- the UCC has been adopted by all 50 states and governs the sale of goods in both commercial and non-commercial contracts. (a), (c), and (d) are incorrect, as they misstate the status and coverage of the UCC. See Hornbook Section 1.7.

When you begin performance to accept a unilateral contract, the offeror (under Brackenbury v. Hodgkin): A) must hold the offer open to allow you to complete performance. B) must hold the offer open for a reasonable period of time to allow you to complete performance. C) can revoke the offer at any time prior to the completion of performance. D) can be required to grant an option to hold the offer open, if the value of the offer exceeds $5,000.

B -- the offeror must hold the offer open for a reasonable period of time to allow you to complete performance under the decision in Brackenbury v. Hodgkin. (a) is incorrect, the offeror is not required to hold the offer open for an indefinite period of time to allow completion of performance. (c) is incorrect, as it represents the holding in Petterson v. Pattberg, not Brackenbury v. Hodgkin. (d) is incorrect, as it posits criteria that are not the law and have not been advocated by any court. See Brackenbury v. Hodgkin.

What is required before a court will enforce a contract for services where the price term is not provided in the contract but has been left to be agreed upon by the parties at a later time? A) A fair market price for the services must exist. B) The parties' actions must conclusively show that they have intended to conclude a binding agreement. C) A course of dealing between the parties providing a basis to determine the price. D) The parties had a reasonable justification for not specifying the price at the time of contracting.

B -- the parties' performance under the contract and/or their statements at the time of contracting must show their intent to enter into a binding contract. (a) is incorrect, the UCC gap-filler for price terms does not apply to contracts controlled by the common law (n.b., the common law controls, as this is a contract for services). (b) and (c) are incorrect, as they introduce principles of equity that courts have not adopted to address service law contracts where the price term has been left to be agreed upon at a later time. See Oglebay Norton v. Armco.

Which of the following is best description of the three essential elements of consideration? A) (1) Money (or property), (2) intent to give value and (3) a legal agreement. B) (1) Legal sufficiency, (2) bargained-for exchange, and (3) determinable promises. C) (1) Legal value, (2) objectively induced monetary exchange and (3) reasonable expectation of gain. D) (1) Expression of intent, (2) detrimental reliance on the integrity of the promise and (3) anything that might have value to the reasonable person.

B -- the three essential elements of consideration are bargained-for exchange, legal sufficiency, and determinable promises (that is, promises that are capable of being objectively determined). See Hornbook Section 4.2.

Which one of the following statements about a modification is correct? A) A valid modification can be retracted. B) A waiver can be retracted. C) A modification can never act as a waiver. D) Both of the statements in choice A and choice B are correct.

B -- waivers can be retracted. The other statements are incorrect. See Hornbook Section 5.14.

Under UCC 2-209, a merchant may supply a form which requires modification or rescission be in a signed record. Which of the following is true under the UCC if an oral modification or rescission is made in violation of a clause forbidding any oral modifications of the written contract? A) The attempted oral modification or rescission voids the contract. B) The attempted oral modification can operate as a waiver. C) The attempted oral modification must be supported by consideration. D) The attempted oral modification must be fair and equitable under the circumstances.

B -- while the clause forbids orally modifying the contract, an attempt at oral modification can operate as a waiver. The other statements are incorrect. See Hornbook Section 5.14(b)

Alice cannot find her $800 set of china table settings. Some months later, while at a party for connoisseurs of fine china, and after consuming an adult beverage, Alice proclaims in front of the entire group that she will gladly pay $2,000 to anyone who locates and returns the china to her. Percy locates and returns the china to Alice and demands $2,000. Alice refuses to pay. Percy files suit to recover the $2,000. It is most likely that Percy will: A) prevail, because Percy was a connoisseur of fine china and Alice should have reasonably known that he would be induced to benefit from this bargain. B) not prevail, because there was no meeting of the minds and Alice did not intend to enter into a contract. C) prevail, if a reasonable person would conclude from the statement Alice made that she intended to enter into a contract. D) not prevail, because Percy knew or should have known that Alice was overstating the value of the china.

C -- The "objective theory" of contracts (the majority rule) is that the subjective intent of the parties is not relevant when determining whether a party has given assent to enter into a contract. Pursuant to this theory, a party's intent is determined by examining what a reasonable person, standing in the other party's position, would conclude the party intended based on the words, conduct and demeanor of that party. This principle is true regardless of what the party actually intended. -- Section 2.2 of the Hornbook (6th Edition) pages 23-24; pages 26-27 (7th Edition)

Which one of the following statements best describes the earliest time an offeror's written revocation of an offer will take effect under the majority rule? A) When the offeror manifests the intent to no longer enter into the proposed contract. B) When the revocation is sent to the offeree. C) When the revocation is received by the offeree. D) When the revocation is read by the offeree.

C -- The general rule for business communications (including revocations) is that they are effective upon receipt, which is why (b) is incorrect. (a) is incorrect, the offeror's manifestation of intent must be communicated to the offeree to take effect. (d) is incorrect, as the revocation will be effective upon receipt, whether the offeree reads it or not. See Hornbook Section 2.20(d).

Pat agrees to buy Dana's car for $10,000. The contract between Pat and Dana will be: A) void, unless it is in writing. B) governed by the common law. C) governed by the Uniform Commercial Code. D) governed by the common law, unless both parties are merchants, in which case it will be governed by the Uniform Commercial Code.

C -- The general rule is that contracts are governed by the common law. The exception is a contract for the sale of goods, which will be governed by the Uniform Commercial Code (UCC).

A makes an offer to B by e-mail, suggesting that acceptance should be by e-mail response. B accepted by regular mail with a properly stamped and addressed letter. Assuming that a response by regular mail is an improper medium, when under the traditional rule would B's letter acceptance be effective? A) Never because B used an improper and slower medium to communicate his acceptance. B) When put the acceptance in the mail box. C) When received by A. D) When read by A.

C -- The mailbox rule applies ONLY WHEN using an authorized medium of communication, when an unauthorized medium is used it will be effective upon RECEIPT, which is why (b) is incorrect. (a) is incorrect, use of an unauthorized medium still allows a contract to be formed, unless the offer specifies that the authorized medium is the only way that the offer may be accepted. (d) is incorrect, as the communication is effective upon receipt, whether the offeree reads it or not. See Hornbook Section 2.23(a).

A offers to sell her car to B for $10,000, and promises to keep the offer open for the entire weekend. If on Saturday morning, C offers A $11,000 for the car, may A revoke her offer to B and sell the car to C? A) Yes, but she may be liable for damages to B. B) No, because she promised to keep the offer open. C) Yes, because there was no consideration or substitute for consideration supporting her promise to keep the offer open for the weekend. D) No, because under the UCC no consideration is required.

C -- Under both the common law and the UCC, options require consideration (or a consideration substitute) for them to be enforceable. (a) is incorrect, there are no damages available when an offeror validly revokes an offer prior to acceptance. (b) is incorrect, absent a merchant's firm offer (see (d)), a promise to hold an offer open unsupported by consideration (or a consideration substitute) is not enforceable. (d) is incorrect. While a merchant's firm offer under the UCC would not require consideration to be enforceable, there are no facts to indicate that A is a merchant (n.b., always assume non-merchant status unless there are facts indicating that a party is a merchant). Under both the common law and the UCC, options require consideration (or a consideration substitute) for them to be enforceable. See Hornbook Section 2.25.

A makes an offer by mail to B. B sends an acceptance by first class mail, but then sends a rejection by overnight express mail. A receives the rejection first. Which of the following best states the rights of A and B under the Restatement view? A) There is no contract between A and B. B) There is a contract between A and B. C) There is a contract between A and B, but B has offered to rescind it. D) There is a contract between A and B, but at B's option.

C -- Under the mailbox rule, the acceptance is effective upon dispatch, making the rejection merely an offer to rescind (as a contract has already been formed). (b) is also correct, as far as it goes; because (c) also addresses the status of the rejection, it is a more complete and therefore the better answer than (b). (a) is incorrect, as a contract was formed when B's acceptance was dispatched. (d) is incorrect, as B's ineffective rejection does not create an option for B to form or not form a contract. See Hornbook Section 2.23(a).

Retailer placed a notice in a trade journal stating that it had men shoes in various sizes available for sale at a price of $75 per pair. When Customer went to Retailer's store to purchase a pair of men's shoes at the advertised price, Retailer refused to sell Customer the shoes for $75. If Customer brings an action in court, which of the factors listed below will the court LEAST LIKELY consider as important when determining whether Retailer's notice in the trade journal constitutes a valid offer? A) Whether the quantity and quality of the shoes was stated in the notice. B) Whether the language used in the notice is promissory in nature. C) Whether Retailer intended the notice to be an offer to sell shoes to Customer. D) Whether Customer reasonably believed that Retailer's notice was an offer.

C -- Under the objective theory of contracts (the majority rule), a party's intent to enter into a contract will be judged solely based on what that party says or does, rather than any subjective and undisclosed intent. In addition, a notice in a trade journal will generally not be considered as an offer to sell the items listed in the notice unless promissory language is used demonstrating a willingness to be bound and a sufficient description of the quantity and quality of goods offered for sale.

Sir Henry Magazine derives a great deal of income from the sale of advertising. It publishes a "rate sheet" as well as an "Advertising Guidelines" brochure. The Advertising Guidelines brochure states, "Sir Henry reserves the right to refuse to print advertising for untruthful copy." A nonprofit political action group presented an advertisement stating that a local political figure had used political contributions for personal use. The group paid the advertising rate in accordance with the rate sheet, but the magazine refused to print the advertising even though it admitted the advertisement was truthful. Will the political action group prevail against the magazine? A) Yes, because the advertisement was truthful. B) Yes, there was a written offer to provide advertisement at a stated price, which was accepted when the political action group tendered a copy of its advertisement and sufficient funds. C) No, the magazine's rate sheet is only a statement of intention to sell or invitation for offers. D) No, because the political action group could not reasonably rely on the magazine's offer.

C -- When an advertisement does not contain a quantity term or language of commitment, it will only be construed as a preliminary proposal and an INVITATION for offers, rather than an offer. (a) is incorrect, because the magazine had not made a valid offer (likewise, a proposed contract term cannot be enforced if no contract has been formed), but even if it was, there is nothing to indicate that this is the only reason for which the ad could be refused. (b) and (d) are incorrect, as the magazine had not made a valid offer. See Hornbook (6th Edition) page 32-33; pages 35-36 (7th Edition).

A large daily newspaper publishes and distributes two booklets, one regarding general advertising rates, and the other regarding advertisements that will not be accepted. These booklets state that the newspaper will refuse advertising that is "dishonest, indecent or illegal." A labor union presented an advertisement to the newspaper that urged readers not to patronize certain businesses because those businesses featured imported clothing that was manufactured by low-wage, foreign labor. The union also paid the appropriate advertising rate in accordance with general advertising rates, but the newspaper refused to print the advertising. Assume that the advertisement was not dishonest, indecent, or illegal. Will the union prevail against the newspaper for breach of contract? A) Yes, because the advertisement was not dishonest, indecent or illegal. B) Yes, there was a written offer to provide advertisement at a stated price, which was accepted when Plaintiff tendered a copy of its advertisement and sufficient funds. C) No, the Defendant's booklet containing its "General Advertising Rates" is only a statement of intention to sell or invitation for offers. D) No, because the union could not reasonably rely on the newspaper's offer.

C -- When an advertisement does not contain a quantity term or language of commitment, it will only be construed as a preliminary proposal and an invitation for offers, rather than an offer. Note: just because something is not actionable in contract does not mean that a remedy cannot be pursued under other areas of law. (a) is incorrect, as the "dishonest, indecent or illegal" criteria are not the only bases for refusing an offer, if an offer had been made; (b) and (d) are incorrect, as no offer has been made. If you missed this question, please review Section 2.6(c) of the Hornbook (6th Edition) pages 31-32; page 34 (7th Edition).

A contract is called a "hybrid" contract if it involves the sale of both goods and services. What is the most accurate statement regarding the test most courts will apply to determine whether UCC Article 2 or the common law should apply to a "hybrid" contract? A) Whether the contract's language indicates that the parties believed that goods or services were the more important element of their agreement. B) Whether the charge for goods exceeds that for service. C) Whether the contract's predominant factor--the thrust, the purpose, reasonably stated--is the rendition of service, with good incidentally involved; or is a transaction of sale, with labor incidentally involved. D) Whether the nature of the goods in question are movable when completed and delivered to the buyer.

C -- Where a hybrid contract is not divisible, courts will determine the predominant factor in order to decide whether the UCC or the common law will control. (a), (b), and (d) are not correct, as none of them represent the test that the court uses to determine the controlling law in hybrid contract involving goods and services. See BMC Industries, Inc. v. Barth Industries, Inc.

John had read many newspaper and Internet news articles suggesting that the fillings commonly used by dentists to fill teeth contained dangerously high levels of mercury. John decided that he would have all his fillings removed and replaced with another safer alternative. He consulted with Dr. Decay, his dentist, and requested removal of all his fillings. Dr. Decay stated that he was not comfortable performing the oral procedure, but that he would make sure that Dr. Abcess would perform the oral procedure. In Dr. Decay's informed medical opinion, removal of John's fillings is not medically required. If Dr. Decay fails to retain Dr. Abcess to perform the oral procedure, will John have an action for breach of contract against Dr. Decay? A) No, a doctor is not liable in breach of contract for breach of an implied promise to perform medical responsibilities, and such a breach is only compensable in a tort action for malpractice. B) Yes, because retaining Dr. Abcess was an express condition of the agreement. C) No, unless John reasonably concluded that Dr. Decay expressly promised him that Dr. Abcess would perform the procedure. D) No, because Dr. Decay told John that he was not comfortable performing the procedure.

C -- While it is true that a doctor is not ordinarily liable in contract for breach of an implied promise to provide a reasonable level of medical competency, a doctor is liable in contract for failure to honor an express promise, which is why (a) is incorrect. (b) is incorrect, because retaining Dr. Abcess is a promise, not an express condition; (d) is incorrect, because it has no bearing on whether the retention of Dr. Abcess is a condition or a promise. If you missed this question, please review Section 2.6(a) of the Hornbook (6th Edition) page 28-30; pages 31-32 (7th Edition).

In determining whether an advertisement is an offer: A) courts look to whether the text is described as an advertisement. B) courts look to whether the text is communicated to an identifiable offeree. C) courts look to whether the text contains the requirements for a valid offer. D) courts look to whether the text contains a specific dollar figure.

C -- courts ignore the way the text is described and determine whether it contains the requirements for a valid offer, which is why (a) incorrect. (b) and (d) are incorrect, while both represent one of the requirements for a valid offer (n.b.), (d) is only a requirement under the common law), neither describe (either by list or in general terms) all of the requirements for a valid offer, making (c) the better answer. See Carlill v. Carbolic Smoke Ball Co.

Widget Company manufactures widgets and had a number of clients who sold products to consumers. Widget wanted to compete with some of its clients, but believed that its clients would be displeased and might cease doing business with Widget. In order to keep this new business activity secret, Widget set up a subsidiary company to distribute its products and formalized its agreement with Paul in writing, which stated that Paul agreed to purchase all shares of the subsidiary for $10,000 and Widget agreed to sell the shares at that price. Widget and Paul had an oral, side agreement that the stock purchase agreement would not be carried out. The parties carried on business for many years without a transfer of the shares or payment of the money. For all appearances, Paul was the sole owner of the distribution business. However, Paul always acted as a loyal employee of Widget. Widget decided to end its arrangement with Paul and to let it be known to the trade that it was operating the distribution business.Paul tendered $10,000 to Widget and demanded that Widget transfer the shares of stock pursuant to the written agreement. Widget refused and sued Paul for a declaratory judgment to the effect that the written agreement is not binding.Will Widget succeed? A) No, because Widget and Paul entered into a detailed formal written agreement and the parties conducted themselves in a manner such that a reasonable person would conclude that Paul was the owner of the shares. B) No, because an oral agreement that contradicts a written agreement is not valid. C) Yes, because Widget and Paul never intended to enter into an agreement. D) Yes, because it was Widget's product from the start.

C -- Widget and Paul both intended their agreement to have no binding legal effect, and thus understanding was expressed between them. As a result, there is no valid contract and Widget need not sell the shares to Paul. (a) and (b) are incorrect, although these factors could point to a validly formed contract, the fact that the parties intent not to form a contract will control over other factors suggesting contract formation; (d) is incorrect, because it does not address a requirement for contract formation. If you missed this question, please review Section 2.4 of the Hornbook (6th Edition) page 26; page 28 (7th Edition)

A Dead Man's Statute: A) preserves the testimony of deceased persons. B) allows contracts to be enforced after the death of one of the signatories. C) prevents testimony about acts or conversations of the decedent. D) prevents lawsuits from continuing after the death of a party.

C -- a Dead Man's Statute prevents testimony about acts or conversations of the decedent. See Miller v. NBD Bank.

Assume that Bill has a valid claim against Jan for $10,000 due in one year for a loan Bill made to Jan. Later Jan and Bill become good friends and Bill tells Jan she does not have to pay the full $10,000 if she will instead pay him $8,000 within one week. Five days later, Jan pays Bill $8,000. If Bill on year later asserts the breach of contract claim against Jan seeking $2,000 he claims is still owed under the original loan, a court will most likely find that A) Bill's surrender of the $10,000 claim is not supported by consideration because Jan had a pre-existing duty to pay $10,000. B) Bill's surrender of the $10,000 claim is not supported by consideration if Bill and Jan, later, decide not to be friends. C) Bill's surrender of the $10,000 claim is supported by new consideration--the right to receive payment within one week. D) As a matter of law, friendship cannot be the basis for surrendering a claim.

C -- as new consideration provided by both parties, Bill's surrender of a $10,000 claim and Jan's assuming an obligation to pay within one week, which is why (a) is incorrect. (b) and (d) are incorrect, as friendship is not part of the consideration supporting the change in terms. See Hornbook Section 4.9.

Alec wants to sell some china to Ben, but he knows that Ben often does not pay his bills. Cecil, Ben's wealthy brother, tells Alec that he (Cecil) promises to pay if Alec promises to deliver the china to Ben. Alec agrees, but later fails to deliver the china. Ben sues Alec. Is Alec's promise to deliver the china supported by consideration? A) No, because Ben incurred no legal detriment. B) Yes, because Ben knew that Cecil would pay. C) Yes, because legal detriment may be given by someone other than the promisee. D) No, because the promise to deliver is only between Alec and Ben.

C -- because consideration exists to support a promise/performance so long as it is bargained for and given in exchange for the promise/performance made by the promisor. Here, Alec was bargaining for Cecil's promise to pay for the china. Once Cecil promised to pay, Alec's promise to deliver the china to Ben was supported by consideration. It does not matter who suffers the legal detriment, as long as it is the legal detriment the promisor is bargaining for, which is why (a) and (d) are incorrect. (c) is incorrect; while knowledge that someone would pay might encourage someone to enter into a contract, is not a requirement for consideration. See Hornbook Section 4.2(a-c) (6th Edition) page 152; pages 159-160 (7th Edition).

Devlon owned a house that he rented to Paula. Paula was a rowdy sort and continually hosted loud parties, disrupting the neighborhood, all in violation of the lease. Rather than fight Paula in court to get him to leave, Devlon offered to pay Paula's moving expenses if Paula would vacate the premises peacefully and relocate outside of his neighborhood. Paula agreed. Was Devlon's promise to pay Paula's moving expenses supported by consideration? A) Yes, if Paula's lease has not yet expired. B) No, if either of the two promises made by Paula is unenforceable. C) Yes, if either of the two promises made by Paula is valid and enforceable. D) No, because Paula is in violation of the lease and has to leave.

C -- because if only one of two conjunctive promises is enforceable, there is still sufficient consideration to support the return promise. Here, when Paula promised to leave peacefully and relocate outside of the community, Paula made two conjunctive promises. If either promise is enforceable, there is sufficient consideration to support Devlon's promise to pay Paula's moving expenses, which is why (b) is incorrect. (a) is incorrect; (a) has violated the terms of the lease (breaching the contract), meaning that Paula cannot then enforce its terms. (d) is incorrect, the fact that Paula is in violation of the lease and has to leave does not prevent Paula vacating the premises peacefully and relocating outside of the neighborhood from serving as consideration. See Hornbook Section 4.14 (6th Edition) pages 191-192; pages 199-200 (7th Edition).

Which one of the following statements is most accurate? A) Under the UCC, a modification must be express. B) Under both the UCC and common law, modification can be implied only if modification was reasonably contemplated at the time of the contract. C) Under both the UCC and common law, modification may be either express or implied. D) Under the UCC and common law, modification may be implied only where there are economic exigencies.

C -- both the UCC and the common law allow express and implied modifications without restriction to specific circumstances; because of this, both (a) and (d) are incorrect. (b) is incorrect, neither the UCC nor the common law impose such a restriction. See Hornbook Section 5.14.

Where a court must gap-fill a term not articulated in the UCC: A) the contract will be void, as only contracts under the UCC can be gap-filled. B) the contract will be void, if the term to be gap-filled is a material term. C) the court should take into account (in this order) the intention of the parties, the nature and purpose of the contract, good faith and fair dealing, and reasonableness. D) the court must take into account the practices of the trade or industry in question.

C -- courts should take into account (in this order) the intention of the parties, the nature and purpose of the contract, good faith and fair dealing, and reasonableness in gap-filling contracts outside of the gap-fillers provided under the UCC. (a) is incorrect, as contracts under the common law can also be gap-filled. (b) is incorrect, as material terms can be gap-filled. (d) is incorrect, as it does not consider the primary role of the intent of the parties and their purpose in contracting. See Hornbook Section 2.9(a)(2).

An illusory promise: A) is enforceable, if it is made in good faith. B) must be supported by consideration. C) is unenforceable. D) is enforceable, if it is in writing.

C -- illusory promises are not enforceable. (a) and (d) are incorrect, because there is no way to make an illusory promise enforceable. (b) is incorrect, because a promise that is supported by consideration is not illusory. See Strong v. Sheffield.

Dick entered into an employment agreement with Ajax Company that provides for a set salary and also for a performance-based bonus. Ajax paid Dick his salary for the first year, but refused to pay the performance-based bonus stating that the consideration for Dick's work was the salary and there was no consideration for the bonus - there must be a separate consideration for the bonus. Dick brought suit against Ajax for the bonus. Who will prevail in this lawsuit? A) Dick, because Ajax acted in bad faith. B) Ajax, because the salary was in exchange for Dick's work so there was no consideration for the bonus. C) Dick, because one consideration will support many promises. D) Ajax, because there is no evidence that the bonus was earned.

C -- one consideration can support many promises, provided that the one consideration induced all of the promises. See Hornbook Section 4.15.

A contract in which one or more of the parties has the power to terminate the legal relations created by the contract is best referred to as: A) an implied in fact contract. B) an implied in law contract. C) a voidable contract. D) a quasi-contract.

C -- such contracts are called voidable. (a) and (b) are incorrect, as they both refer to methods of communication used in forming contracts and do not speak to whether a contract can be terminated by one or both parties; (d) is incorrect, as quasi-contract is an equitable remedy imposed by a court in order to prevent unjust enrichment. See Hornbook Section 1.8(c).

When you begin performance to accept a unilateral contract, the offeror (under Petterson v. Pattberg): A) must hold the offer open to allow you to complete performance. B) must hold the offer open for a reasonable period of time to allow you to complete performance. C) can revoke the offer at any time prior to the completion of performance. D) can be required to grant an option to hold the offer open, if the value of the offer exceeds $5,000.

C -- the offeree can revoke the offer at any time prior to the completion of performance under the decision in Petterson v. Pattberg. (a) and (b) are incorrect; while they represent possible results if the approach outlined in the dissent to Petterson v. Pattbergwere followed, they do not represent the holding of the majority in Petterson v. Pattberg. (d) is incorrect, as it posits criteria that are not the law and have not been advocated by any court. See Petterson v. Pattberg.

Which of the following is not an accurate statement with regard to course of performance and its effect on modification? A) Course of performance may constitute a waiver of Statute of Frauds defense. B) Course of performance may be a proper basis to support an oral modification of a contract within the UCC Article 2 Statute of Frauds. C) Course of performance may provide the consideration a court can consider under both the common law and UCC Article 2. D) Course of performance may be the basis for enforcement of a modification even if not included in a signed writing.

C -- the parties' course of performance may provide the consideration a court can consider under both the common law and UCC Article 2 is NOT an accurate statement; the other answer choices are accurate statements of the law. See Hornbook Section 5.14(a).

Between the UCC and the common law, the controlling law of a contract is determined by: A) The controlling law chosen by the parties in their written contract. B) The parties' status as merchants or non-merchants. C) The subject matter of the contract. D) The court, in the event of litigation.

C -- the subject matter of the contract determines which law is controlling. (a) and (d) are incorrect, as the controlling law cannot be chosen by contracting or by the court. (b) is incorrect, as merchant status (which only exists under the UCC) does not determine the controlling law. See BMC Industries, Inc. v. Barth Industries, Inc. and Hornbook Section 1.7.

Buyer and Seller want to do business together, but they would like to have a relatively informal arrangement. They prepare a written "Gentlemen's Agreement," but state in the agreement that it is not binding. It also states, "This writing is only an expression of our intention to work honorably together. We have always worked well together, therefore, we do not wish to be bound by this document." When the business relationship ends there are $100,000 worth of orders pending, but not delivered by Seller. Seller refuses to ship the orders and Buyer sues. Does Buyer prevail? A) Yes, because Seller had accepted the orders. B) Yes, because a "gentlemen's agreement" is not valid. C) No, because this was just a "gentlemen's agreement" and was not intended to have legal effect. D) No, because one cannot create an agreement with a clause that states the agreement is not valid.

C -- when the parties EXPRESSLY state that they do not intend to be bound by their so called "gentlemen's agreement," the courts conclude that a contract does not arise. (even though orders were pending, it's of no matter). (a) is incorrect, the actions of the parties do not negate their intent to not form a contract. (b) is incorrect, a "gentlemen's agreement" can be valid. (d) is incorrect, because it is possible to create just such an agreement (as was done here). See Hornbook (6th Edition) page 26; page 28 (7th Edition).

Arthur calls Betty and says: "I am in a real fix and I must sell my house. I am so motivated that I would consider $100,000 for it." Immediately Betty says: "I'm sorry about your predicament. I will buy your house for $100,000." Is there a contract? A) No, because some of the material terms are missing. B) Yes, because Arthur clearly intended to sell his house for $100,000 and Betty accepted. C) No, because Arthur never accepted Betty's offer. D) Yes, because Betty accepted by agreeing to the price terms stated by Arthur.

C -- while Arthur stated that Arthur "must sell" and "would consider" taking $100,000, Arthur never made an offer to sell the house to Betty. Arthur's statements were merely preliminary negotiations and were not intended as an offer to sell the house to Betty for $100,000. Betty made an offer by her statements and Arthur must accept in order for formation. (a) is incorrect; many material terms can be gap-filled, allowing a contract to be formed where the intent to form a contract is clear (as is the case here). (b) and (d) are incorrect, because Arthur has not made a valid offer that would give Betty the power of acceptance. See Hornbook (6th Edition) pages 38-40; pages 41-43 (7th Edition).

If Bob and Sally are friends and Sally promised to sell her television to Bob for $50 where the actual value of the television is $1,000, is there proper consideration to support Sally's promise? A) No, because $50 dollars is not a legal detriment. B) Yes, because the friendship of Bob and Sally is not relevant. C) No, if the primary motivation for the exchange is their friendship, D) Yes, if there is a bargained-for exchange in their agreement.

D -- Consideration requires a bargained-for exchange that is legally sufficient; there is no requirement that the amount of consideration reflect the market value of the property or a fair price, which is why (a) is not correct. While (B) is accurate in that the friendship of the parties is not relevant (which is why (c) is incorrect), that information does not address why (B) (yes) is correct; however, (d) explains why it is correct, making it the best answer choice. See Hornbook Section 4.7.

Which of the following factors will a court consider when determining whether the parties have entered into a valid modification of an existing contract? A) Duress B) Statute of Frauds C) Good Faith D) All of the above

D -- Duress, good faith, and the applicability of the requirements of the Statute of Frauds are all considered in determining whether a valid modification has been made to an existing contract. See Hornbook Section 5.14.

A, a merchant, makes an offer by mail to B in a signed writing. The offer provides assurances that it will be held open for two weeks. The following week, A changes her mind and calls B to revoke the offer. Is the offer revocable? A) Yes, because there was no consideration for B's promise to hold the offer open. B) Yes, because B is not a merchant. C) Yes, if A has proposed a fair exchange. D) No.

D -- Once given, a merchant's firm offer is irrevocable for the period stated. (a) is incorrect, consideration is not required for a merchant's firm offer to be irrevocable. (b) is incorrect, only the party granting a merchant's firm offer must be a merchant (i.e., a merchant's firm offer can be granted to anyone). (c) is incorrect, fairness of the exchange is not required for a merchant's firm offer to be irrevocable. See Hornbook Section 2.25(a).

Which of the following correctly states the Restatement (Second) definition of offer? A) A promise to do or refrain from doing some specified thing in the future conditioned on the other party's acceptance. B) A promise that, according to its terms, is contingent upon a particular act, forbearance, or promise given in exchange for the original promise or the performance thereof; made in such a way that another individual is justified in understanding that his or her assent to the bargain is invited and that such assent will conclude the bargain. C) A promise of willingness to contract on certain terms, made with the intention that it shall become binding as soon as it is accepted by the person to whom it is addressed D) The manifestation of willingness to enter into a bargain, so made as to justify another person in understanding that his assent to that bargain is invited and will conclude it.

D -- Section 2.6(a) of Hornbook.

In an auction, generally the auctioneer:I. makes an offer, which the bidder can accept or reject.II. makes an invitation to bid, then the bidder makes the offer.III. can accept the offer.IV. can seek another offer. A) I. and III. B) II. and IV. C) III. and IV. D) II., III., and IV.

D -- Section 2.6(e) of Hornbook.

In most modern jurisdictions is a contract whereby the parties promise to negotiate an agreement enforceable? A) No, because if the parties fail to agree there would be no remedy. B) No, because there would be no consideration. C) Yes, because the court could determine what a reasonable agreement would be. D) Yes, because the parties would be obligated to negotiate in good faith.

D -- The obligation to negotiate in good faith enables a promise to negotiate to be enforceable. (a) and (b) are incorrect, as they are inaccurate statements of the law. (c) is incorrect, because the court will not substitute its judgment for that of the parties. See Copeland v. Baskin Robbins, where the specific context of the contract and the continuing business relationship between the parties distinguishes this from the usual treatment of an agreement to agree as not enforceable..

Offeror posts on a Web page an offer that can be accepted by any viewer. If the offeror wants to revoke the offer, the offeror must do which of the following? A) Must wait until a reasonable time has lapsed. B) Must inform each offeree that the offer has been revoked. C) Need not do anything as offers cannot be posted on the internet. D) Must post a revocation of the offer on the Web page.

D -- To be effective, a revocation of a public offer must be communicated in the same manner as the public offer (in this case, webpage posting). (a) is incorrect, there is no time delay requirement. (b) is incorrect. This could only be an effective way of revoking a public offer, if you could determine which members of the general public were aware of the public offer (which is impossible in most circumstances). (c) is incorrect, public offers posted on the internet can be accepted. See Hornbook Section 2.20(d).

A party is a merchant under the UCC if he or she: A) deals in goods of the kind involved in the transaction. B) by occupation holds himself or herself out as having knowledge or skill peculiar to the practices involved in the transaction. C) hires as his or her agent someone who qualifies as a merchant under (a) or (b). D) all of the above, (a), (b), or (c).

D -- UCC 2-104 provides that persons who meet the any of requirements under (a), (b), or (c) are merchants. See Hornbook Section 1.7.

You have lunch to go from the same place every day, and you always order the same thing, your "usual." You go to the counter and the cashier hands you your usual, neither of you speak a word. Which of the following is correct? A) You have received unsolicited merchandise, you are not required to pay for it. B) By conduct, the cashier has solicited an offer from you. C) By conduct, the cashier has made an offer which you may accept or reject. D) By conduct, you have made an offer and the cashier has accepted it.

D -- by your conduct you have made an offer, and the cashier (also by conduct) has accepted it. (a), (b), and (c) are incorrect, as your conduct is a valid offer. See Hornbook Section 2.19.

Anne and Bob were co-owners of a small, very successful restaurant. Anne was interested in opening a second location in a nearby town and asked Sam (a regular customer of the restaurant) if he would be willing to sell Anne and Bob a particular parcel of land Sam owned in the nearby town of Greenville for $100,000. Sam said "Sounds great...let's do it." However, unknown to Anne and Bob, Sam had serious doubts about selling the land and was considering developing the land himself to build and operate a restaurant. When Anne attempted to hand Sam the $100,000, Sam refused to take the money and told Anne he would not sell Anne and Bob the land. Anne and Bob sued Sam for breach of contract. Sam alleged he never intended to sell the land to Anne and Bob.At trial, the following three statements were offered: (1) Bob testified that he and Anne had "no definite plan to ever open a second restaurant, and that Sam was aware of this." (2) Sam testified that he "never really intended to sell the property to Anne and Bob." (3) Anne testified that she was very surprised when Sam so quickly agreed to sell because Sam was always joking about "how you should buy my land in Greenville and open a second restaurant there so I wouldn't have to drive so far for breakfast."Which of the statements will the court consider important to determine if a valid contract was formed? A) All of the statements. B) Statement 1 and 2 only. C) Statements 2 and 3 only. D) Statements 1 and 3 only.

D -- Under the "objective theory" of contracts, the subjective intent of either party is not a factor when determining whether the parties were serious about intending a contract or that a contract was formed. Instead, it is the objective, reasonable interpretation by someone in the position of the contracting parties (based on their words and conduct) that will determine whether the intent to enter into a contract exists. Thus, the evidence in statement 2 that Sam "never intended to sell the property to Anne and Bob" would not be important because that is merely the subjective undisclosed intent of a party. However, both statements 1 and 3 would be important. Statement 1 is evidence that Bob and Anne were not seriously planning to open a second restaurant and thus unlikely (as reasonable parties to a potential agreement) to consider any statement by Sam as an "acceptance" of any inquiry made regarding the possible purchase of his parcel of land. Statement 3 is also important because it shows that a reasonable person in Anne and Bob's position would not have interpreted Sam's statement as an offer because he was "always joking" about selling the land to them. If you missed this question, please review Section 2.2 of the Hornbook (6th Edition) pages 23-24; pages 26-27 (7th Edition).

A makes an offer by mail to B. B sends a rejection by mail, but before A receives it, B sends an acceptance. A receives B's rejection before receiving B's acceptance. Under the Restatement view there is: A) a contract when the acceptance was sent. B) a rejection of the offer when the rejection was read. C) a rejection of the offer when the rejection was sent. D) a rejection of the offer when the rejection was received.

D -- Under the Restatement approach, when the rejection arrives FIRST, it is effective when received. (a) is incorrect, when both a rejection and an acceptance have been dispatched, the acceptance is not effective until it is received. (b) is incorrect, as the rejection is effective upon receipt, whether the offeree reads it or not. (c) is incorrect, as the rejection is effective upon receipt. See Hornbook Section 2.23(a).

How can an offer to buy goods be effectively accepted? A) only by shipping the goods. B) only by promising the ship the goods. C) by either a shipment of conforming goods or a promise to ship the goods. D) by either a shipment of conforming or non-conforming goods or a promise to ship the goods.

D -- all three methods of acceptance listed are effective under the UCC. (note that nonconforming goods can be dealt with by buyer/offeror after)

Where there is no language in a contract addressing duration, the contract will: A) be void, as it is missing a material term. B) be perpetual in duration. C) be at-will, with either party being able to terminate it whenever they choose. D) run for a reasonable time.

D -- as it allows the court to determine the length of the contract based on the intent of the parties. (a) is incorrect, as duration is not a material term; (b) and (c) are incorrect, as they both provide outcomes that are not able to be adjusted to reflect the intent of the parties. See Haines v. City of New York.

Jill says to Dave, "If you ride your bike from San Diego to New York City by September, I promise to pay you $10,000." Dave promises to ride. Is there a contract? A) Yes, because this is a bilateral contract. B) No, because the terms of the offer may not be reasonable. C) Yes, because Dave promised to ride his bike as agreed in response to Jill's offer. D) No, because Dave did not accept Jill's offer.

D -- because in order to accept Jill's offer, Dave must do the act requested by Jill, not simply promise to perform as requested. (a) and (c) are incorrect, because this is a unilateral contract (not a bilateral contract) and acceptance can only be made by performance (not by promising to perform). (b) is incorrect, the reasonableness of the terms is irrelevant to whether a contract can be formed. See Hornbook Section 2.10(a) (6th Edition) pages 56-58; pages 61-63 (7th Edition).

Dexter and Posley enter into an agreement whereby Dexter is to grow and deliver corn to Posley at a specified time, price, quantity and quality. The agreement states that "either party may terminate this agreement at any time." After more than 10 years of performance under the contract, Dexter terminates the agreement. Is Dexter justified? A) Yes, regardless of whether Dexter gave Posley reasonable notice of termination. B) Yes, because the termination provision was clear and unequivocal. C) No, because a termination clause must be set forth with specificity. D) No, unless Dexter gave Posley reasonable notice.

D -- because the courts will imply a reasonable notice requirement in such contracts to avoid finding the contract is unenforceable due to an illusory promise. Here, the contract is not unenforceable because the parties have performed under the contract for over ten years. Thus, the court would imply that either party must provide reasonable notice before terminating the contract, which is why (a) is incorrect. (b) is incorrect, because that interpretation of the termination clause would render the promise illusory and the contract unenforceable (and the reason why courts imply a requirement of reasonable notice). (c) is incorrect, termination clauses can be set forth with specificity, but they can also be implied. See Hornbook Section 4.12(b)(4-5) (6th Edition) pages 179-183; pages 187-191 (7th Edition).

If an offer has no quantity term: A) no contract can be formed. B) the court will look to the context of the offer to determine a reasonable quantity. C) but does commit the purchaser to buy the amount the purchaser requires, a contract can be formed. D) both a. and c.

D -- both a & C -- The general rule is that no contract can be formed, without quantity (a); however, courts have recognized an exception to this rule where the offer is to purchase all of a certain good that is "required," (c). (b) is incorrect, as courts will not impose quantity terms where there is no express language indicating either an amount or the intent of the parties for the contract to be for the amount required. See Sylvan Crest Sand & Gravel Co. v. United States.

Offeror mailed a valid offer to an intended offeree and then died of a heart attack the next morning. The offeree received the offer at noon and, not knowing of the offeror's death, mailed an acceptance of the offer that same afternoon. Is there a valid contract under the majority view? A) Yes, because an offer is irrevocable for a reasonable time once mailed. B) Yes, if the offer is not one for personal services that only the offeror could personally perform. C) Yes, because the offeree did not know or have reason to know of the offeror's death when the acceptance was mailed. D) No, the offeror's death terminated the offeree's power of acceptance.

D -- death of an offeror terminates an offer that has not been accepted (a) is incorrect, mailing does not make an offer irrevocable. (b) is incorrect, regardless of the subject matter, all offers that have not been accepted terminate upon the death of the offeror. (c) is incorrect, the offeree's knowledge of the death of the offeror has no impact of the offer's termination upon the death of the offeror. See Hornbook Section 2.20(c).

Benson owned a golf course. Benson was approached by the Magson Railroad Company, that asked for a right of way across Benson's land that was adjacent to his golf course. Benson agreed to the right of way in exchange for the railroad's promise to build "a really cool railway station" at a specified spot near the golf course. The railroad was built on the right of way, but Magson did not build the station. Benson sues Magson for the station. Will Benson prevail in this lawsuit? A) No, because this was an agreement for the transfer of an interest in real property. B) No, because the agreement was never memorialized in a writing. C) Yes, because Magson must build the station to mitigate his damages. D) Yes, because the parties agreed that Magson would build a station near the golf course in exchange for the right of way.

D -- expresses the intentions of the parties to enter into a contract as well as its specific terms. (a) and (b) are incorrect, as neither of these issues have any bearing on whether the agreement would be enforceable. (c) is incorrect, because the concept of mitigation of damages impacts the amount of damages awarded, not whether a contract is enforceable. See Hornbook (6th Edition) pages 41-43; pages 44-47 (7th Edition).

Jill says to Dave, "If you ride your bike from San Diego to New York City by September, I promise to pay you $10,000." Dave promises to ride. Is there a contract? A) Yes, because this is a bilateral contract. B) No, because the terms of the offer may not be reasonable. C) Yes, because Dave promised to ride his bike as agreed in response to Jill's offer. D) No, because Dave did not accept Jill's offer.

D -- in order to accept Jill's offer, Dave must do the act requested by Jill, not simply promise to perform as requested. (unilateral contract must be accepted by performance, not merely promise to perform) (a) and (c) are incorrect, because this is a unilateral contract and cannot be accepted by a promise. (b) is incorrect, the reasonableness of the terms is irrelevant to determining whether the offer was accepted. See Hornbook (6th Edition) pages 56-58; pages 61-63 (7th Edition).

Unsolicited merchandise A) must be paid for or returned. B) may be kept and not paid for. C) must be paid for, if there is a pre-existing business relationship. D) both b. and c.

D -- includes both the general rule for unsolicited merchandise (b), as well as the exception to the general rule when a business relationship exists (c). (a) is incorrect, as it contradicts 39 USC §3009. See 39 USC §3009 (for the general rule) and Hobbs v. Massasoit Whip Co. (for the exception).

In a sale of goods contract, how will the courts respond to a failure of the parties to specify a quantity of goods? A) Courts will use one of the gap-filler provisions of the UCC to fill in the missing term. B) Courts will use standard practices in the relevant trade or profession to fill in the missing term. C) Courts will use a reasonableness standard to fill in the missing term. D) Courts will not enforce the contract because it is missing a material term

D -- quantity is a contract term that neither the UCC nor the common law will gap-fill. See Hornbook Section 2.9(a)(2).

If an offer is unclear as to the proper mode of acceptance, how can the offer be accepted? A) Only in the same manner used to communicate the potential offer. B) Only by a promise to perform. C) Only by performance of the requested act. D) In any manner reasonable under the circumstances.

D -- states the law accurately. (a) is incorrect, as it addresses methods of communication, not the proper mode of acceptance, and is too narrow in any event; (b) and (c) are incorrect, as they do not accurately state the proper modes of acceptance when an offer is unclear as to the proper mode of acceptance. See Hornbook Section 2.10(a).

Under UCC 2-209, a merchant may supply a form which requires modification or rescission be in a signed record. Which of the following is a true statement where the contract containing that clause is contained a contract between a merchant and a non-merchant? A) The clause only becomes effective once the contract is signed by both parties. B) A clause forbidding oral modification is not permissible in a contract between a merchant and a non-merchant. C) The clause becomes effective once the merchant signs the contract. D) The clause must be separately signed by the non-merchant, otherwise the clause is ineffective.

D -- the clause must be separately signed by the non-merchant to be effective. None of the other statements are true with regard to such a clause in the context of a contract between a merchant and a non-merchant. See Hornbook Section 5.14.

Under the traditional rule, an employee who has an employment-at-will contract may be terminated "for good cause, for no cause or even for cause morally wrong." Today, many jurisdictions: A) have made no change to the traditional rule and continue to allow an at-will employee to be terminated for any reason. B) protect at-will employees from no cause termination only if such protection is expressly included in the employment contract. C) only permit termination of at-will employee if for good cause. D) protect at-will employees from no cause termination only if terminated for reasons that are contrary to public policy.

D -- the modern majority approach is to prohibit terminations for reasons that are contrary to public policy. (a) is incorrect, the traditional rule is no longer the modern majority approach. (b) and (c) are incorrect, as what they describe is not employment-at-will. See Hornbook Section 2.9(a)

The offeree accepts the offer: A) when the offeree acts affirmatively. B) when the offeree acts with the intent to accept the offer. C) when the offeree acts to do what the offer requires for acceptance. D) both b. and c.

D -- the offeree must do what the offer requires for acceptance (c), whether that is performance or promise, and must do so with the intent to accept the offer (b). (a) is incorrect, as it does not describe what the offeree must do to accept an offer. See Hornbook Section 2.13.

Paul owes Big $10,000. Since Paul has lost his job and cannot afford to pay Big the entire $10,000 owed, Paul wrote a check to Big for $1,000 - all the money that Paul has - and wrote on the back of the check, "Accept only as payment in full of my $10,000 debt." Big endorsed and cashed the check. Which of the following is the most accurate statement of Paul's obligation to Big? A) Paul does not owe anything further to Big since Big cashed the check in spite of the endorsement on the back of the check. B) Paul still owes $9,000, the balance of the debt, to Big because writing this language on the back of a check will not relieve a debtor of the balance of the debt. C) Paul does not owe the $9,000 balance of the debt to Big because Big cannot reasonably expect Paul to pay the balance of the debt because Paul has lost his job. D) Paul still owes the $9,000 balance of the debt to Big if there is no good faith dispute between Paul and Big regarding the amount owed or if Big was not aware of the basis of the dispute.

D -- unless there was a good faith dispute over the amount owed by Paul, Big's acceptance of $1,000 to settle it could not serve as valid consideration, which is why (a) is incorrect. (b) is correct but not the best answer, because it does not explain the law relating to settlement language on check endorsements as completely as (d). (c) is incorrect, as ability to pay does not alter the terms of a contract. See Hornbook Section 4.11(c).

When you begin performance to accept a unilateral contract, under Motel Services, Inc. v. Central Maine Power Co. the offeror: A) must hold the offer open to allow you to complete performance. B) can alter the terms of the offer, if those terms do not involve the material provisions of the offer and have a dollar value of $500 or less. C) can revoke the offer at any time prior to the completion of performance. D) cannot alter the terms of the offer.

D -- while the offeror is holding the offer open for a reasonable time to allow you to complete performance, the offeror cannot alter the terms of the offer under Motel Services, Inc. v. Central Maine Power Co. (a) is incorrect, the offeror is not required to hold the offer open for an indefinite period of time to allow completion of performance. (b) is incorrect, as the offeror cannot alter the terms of the offer under Motel Services, Inc. v. Central Maine Power Co. (c) is incorrect, as it represents the holding in Petterson v. Pattberg, not Motel Services, Inc. v. Central Maine Power Co. See Motel Services, Inc. v. Central Maine Power Co.


Related study sets

MIEH331 Built Enviro Final Exam Guide

View Set

The Outsiders Chapters 7 - 9 Vocabulary

View Set

Care of the postpartum family --> OB exam 2

View Set

Traumatic Injuries of the Musculoskeletal System

View Set

success strategies- Jessica Silnutzer

View Set

polisci ch 22: (califonrnia government) the governor and the executive branch

View Set